0 Daumen
293 Aufrufe

Geben Sie zu den folgenden Potenzreihen bezüglich der allgemeinen Form \( \sum \limits_{k=0}^{\infty} a_{k}\left(x-x_{0}\right)^{k} \) jeweils die Koeffizienten \( a_{k}, \) die Entwicklungsstelle \( x_{0} \) und den zugehörigen Konvergenzradius \( \rho \) an:
(a) \( 12 x+\frac{4 !(1+2 x)^{2}}{2^{2}}+\frac{5 !(1+2 x)^{3}}{3^{3}}+\frac{6 !(1+2 x)^{4}}{4^{4}}+\cdots \)
(b) \( 5 ! \cdot 3^{3} \cdot\left(\frac{x}{4}-2\right)^{3}+5 ! \cdot 4^{4} \cdot\left(\frac{x}{4}-2\right)^{4}+5 ! \cdot 5^{5} \cdot\left[\frac{x}{4}-2\right)^{5}+\dots \)
(c) \( \sum \limits_{n=3}^{\infty}\left(x \cdot \sin \frac{2}{3 n}\right)^{2 n} \)
(d) \( \sum \limits_{m=2}^{\infty} \frac{m-1}{6+m} \cdot(2 x+2)^{3 m+4} \)


Problem/Ansatz:

zu a): wie kann ich umformen um auf die allgemeine form zu kommen? x0 müsste ja -1/2 sein wenn ich mich nicht irre...
hatten ein beispiel dazu das ich nachvollziehen kann allerdings mit x als 1. summand und nicht 12x und mit 1! im 1. zähler und nicht 4!. das verwirrt mich irgendwie...
zu b): bekomme ich \( \sum\limits_{n=3}^{\infty}{5!(n/4)^n} \) * (x-8)^n raus sowie q=∞ und ρ=0 also konvergiert es nur für x0 =8 ..
zu c) und d) fehlt mir wieder jeglicher ansatz und ich wäre für einen lösungsweg sehr dankbar :(

Avatar von

Ein anderes Problem?

Stell deine Frage

Willkommen bei der Mathelounge! Stell deine Frage einfach und kostenlos

x
Made by a lovely community